Re:[obm-l] Isometria

2007-05-19 Por tôpico claudio.buffara
Como eu disse, T(x,y) = (x,y+1/2) eh apenas uma realizacao, ou seja, apenas um exemplo de isometria que se encaixa no seu contra-exemplo. A minha demonstracao de fato prova que QUALQUER QUE SEJA A ISOMETRIA T: B(0,1) - R^n, se T(0) 0, entao existe r 1 tal que: para todo b em B(0,1) com r

Re:[obm-l] Isometria

2007-05-18 Por tôpico claudio.buffara
-- Cabeçalho original --- De: [EMAIL PROTECTED] Para: obm-l@mat.puc-rio.br Cópia: Data: Thu, 17 May 2007 05:45:33 -0300 (BRT) Assunto: Re:[obm-l] Isometria Ola Claudio. De fato,T(b) e T(-b) sao simetricos em relacao a T(0). O meu contra exemplo mostra que apesar disso ser

Re:[obm-l] Isometria

2007-05-16 Por tôpico claudio.buffara
Oi, Rivaldo: Voce admite que se T eh isometria, entao: T(b) e T(-b) sao simetricos em relacao a T(0)? Soh pra facilitar, repito aqui a demonstracao: Seja T(0) = a. Seja b um ponto qualquer de B. O simetrico de b (em relacao a 0) eh -b. Entao: |T(b) - a| = |T(b) - T(0)| = |b - 0| = |b| (*)

Re:[obm-l] Isometria

2007-05-08 Por tôpico claudio.buffara
-- Cabeçalho original --- De: [EMAIL PROTECTED] Para: obm-l@mat.puc-rio.br Cópia: Data: Tue, 8 May 2007 01:59:26 -0300 (BRT) Assunto: [obm-l] Isometria Seja B={x em IR^(n+1)/ ||x||1} e T: BB uma isometria. Provar que T(0)=0. Se T(0) = a 0, entao considere os

[obm-l] Re: [obm-l] Lista Análise 2005

2007-04-19 Por tôpico claudio.buffara
Antes de mais nada, basta olhar pra x em [0,2pi), pois ao somarmos um múltiplo inteiro de 2pi a x, a série não se altera. Nesse caso, eu acho que essa série só converge se x = 0 ou x = pi. Em ambos os casos, a série é SOMA(n=1) 1/n^(1,8), que é convergente. Para x em (0,pi) união (pi,2pi),

[obm-l] Off-Topic mas nem tanto...

2007-04-10 Por tôpico claudio.buffara
Pro pessoal de exatas... LEITE Por que o queijo e a manteiga são derivadas do leite? Porque o leite é integral! - - - - - - - - - - - - - - - - - - - - - - - - - - - - - - - - - - - JESUS Jesus disse a seus apóstolos: - Irmãos, y = ax²+bx+c... Os apóstolos, confusos, responderam: -

Re: RE: [obm-l] tabuleiro

2007-04-05 Por tôpico claudio.buffara
(m+1) + 4m + 8 -2 E como a fórmula funciona para m=1 (tabuleiro 2x2) e m=2(tabuleiro 4x4) funciona para todos, certo? SDS JG [João Gilberto Ponciano Pereira] -Original Message- From: [EMAIL PROTECTED] [mailto:[EMAIL PROTECTED] Behalf Of claudio.buffara

Re: RE: [obm-l] tabuleiro

2007-04-05 Por tôpico claudio.buffara
=2(tabuleiro 4x4) funciona para todos, certo? SDS JG [João Gilberto Ponciano Pereira] -Original Message- From: [EMAIL PROTECTED] [mailto:[EMAIL PROTECTED] Behalf Of claudio.buffara Sent: Tuesday, April 03, 2007 6:11 PM To: obm-l Subject: Re:[obm-l

Re:[obm-l] tabuleiro

2007-04-03 Por tôpico claudio.buffara
De:[EMAIL PROTECTED] Para:obm-l@mat.puc-rio.br Cópia: Data:Mon, 2 Apr 2007 21:25:39 -0300 Assunto:[obm-l] tabuleiro Alguém poderia me ajudar com essa? Guilherme escreveu um número em cada casa de um tabuleiro 8 x8 (64 casas), de modo que a soma dos números das casas vizinhas de cada

RE: [obm-l] tabuleiro

2007-04-03 Por tôpico claudio.buffara
? SDS JG [João Gilberto Ponciano Pereira] -Original Message- From: [EMAIL PROTECTED] [mailto:[EMAIL PROTECTED] Behalf Of claudio.buffara Sent: Tuesday, April 03, 2007 6:11 PM To: obm-l Subject: Re:[obm-l] tabuleiro De:[EMAIL PROTECTED] Para: obm-l@mat.puc

Re: [obm-l] Funcoes

2007-04-01 Por tôpico claudio.buffara
-- Cabeçalho original --- De: [EMAIL PROTECTED] Para: obm-l@mat.puc-rio.br Cópia: Data: Sat, 31 Mar 2007 23:18:46 -0300 Assunto: Re: [obm-l] Funcoes É o conjunto de Cantor? E como voce prova isso? On 3/30/07, claudio.buffara [EMAIL PROTECTED] wrote: Seja f uma

Re:[obm-l] Funcoes

2007-03-30 Por tôpico claudio.buffara
f(1) = f(1-0) = 1-f(0) = 1 f(1/3) = f(1)/2 = 1/2 f(2/3) = f(1-1/3) = 1-f(1/3) = 1-1/2 = 1/2 = f(1/3) == esta funcao nao eh crescente - pode ser no maximo nao-decrescente. Supondo que seja, prosseguimos... 1/3 = x = 2/3 == f(x) = 1/2. f(1/9) = f(1/3)/2 = 1/4 == f(8/9) = 3/4 f(2/9) = f(2/3)/2 = 1/4

Re:[obm-l] Calculo

2007-03-30 Por tôpico claudio.buffara
Analise Matematica, que eh o embasamento teorico do calculo, ou seja, com demonstracoes rigorosas de todos os teoremas. Eu sugiro comecar com os livros do Elon Lages Lima, que sao otimos e baratos. Analise Real, vols. 1 e 2 e Curso da Analise - vols. 1 e 2. Todos publicados pelo Impa. []s,

Re:[obm-l] Funcoes

2007-03-30 Por tôpico claudio.buffara
Seja f uma funcao não-decrescente definida em [0,1] e tal que f(0)=0, f(x/3)=f(x)/2 e f(1 - x)=1 - f(x). Encontre f(18/1991). Mais interessante do que este problema específico é observar que a imagem de f é densa em [0,1] apesar de f ser constante num conjunto de medida integral em

Re: [obm-l] o menor valor

2007-03-28 Por tôpico claudio.buffara
Infelizmente, a maioria das pessoas que usa multiplicadores de Lagrange segue apenas uma receita de bolo, sem ter a menor ideia de por que o metodo funciona. Uma boa explicacao encontra-se no cap. 4 do livro Analise Real - vol.2 do Elon Lages Lima, publicado pelo Impa. No entanto, nesse caso,

Re: [obm-l] Limite

2007-03-27 Por tôpico claudio.buffara
On 3/26/07, Leonardo Borges Avelino [EMAIL PROTECTED] wrote: Calcule o limite: lim [cos(k/x)]^x x-infinito com k constante sem utilizar l'hospital ou série ou equivalência. somente por limites fundamentais.. grato Leonardo Borges Avelino Isso equivale a lim(t-0+)

Re:[obm-l] Homomorfismo sobrejetor

2007-03-27 Por tôpico claudio.buffara
-- Cabeçalho original --- De: [EMAIL PROTECTED] Para: obm-l@mat.puc-rio.br Cópia: Data: Fri, 23 Mar 2007 19:51:51 -0300 (ART) Assunto: [obm-l] Homomorfismo sobrejetor Olá para todos! Estou com o seguinte problema: Seja d um divisor de n. Prove que o homomorfismo

Re:[obm-l] Problema... Olimpiada Argentina

2007-03-27 Por tôpico claudio.buffara
De:[EMAIL PROTECTED] Para:obm-l@mat.puc-rio.br Cópia: Data:Mon, 26 Mar 2007 21:45:28 -0300 Assunto:[obm-l] Problema... Olimpiada Argentina Não consigo resolver: Para cada número natural, n, n diferente de zero, determinar os inteiros a, b, c e d, 0=a=b=c=d, tais que 2^n=a^2+b^2+c^2+d^2.

Re:[obm-l] algebra complexa dos complexos

2007-03-27 Por tôpico claudio.buffara
De:[EMAIL PROTECTED] Para:obm-l@mat.puc-rio.br Cópia: Data:Tue, 27 Mar 2007 15:06:17 + Assunto:[obm-l] algebra complexa dos complexos Sauda,c~oes, Tenho duas apostilas dos anos 70 que tratam dos números complexos: uma do Morgado (minha) e outra do Reinaldo (?) do Impacto que ganhei

[obm-l] Re: [obm-l] RES: [obm-l] Congruênc ia, módulo m

2007-03-26 Por tôpico claudio.buffara
Dois inteiros a e b sao ditos congruentes modulo m quando ambois deixam o mesmo resto quando divididos por m. Isso eh a mesma coisa (verifique!) que dizer que a-b eh divisivel por m. A notacao usual nesta lista eh a == b (mod m). A utilidade do conceito de congruencia vem dos seguintes fatos

Re:[obm-l] Homomorfismo sobrejetor

2007-03-26 Por tôpico claudio.buffara
-- Cabeçalho original --- De: [EMAIL PROTECTED] Para: obm-l@mat.puc-rio.br Cópia: Data: Fri, 23 Mar 2007 19:51:51 -0300 (ART) Assunto: [obm-l] Homomorfismo sobrejetor Olá para todos! Estou com o seguinte problema: Seja d um divisor de n. Prove que o homomorfismo natural

[obm-l] Re:[obm-l] Método da marcha ré

2007-03-23 Por tôpico claudio.buffara
Este metodo eh muito util quando, por exemplo, o seu carro (C) estah estacionado (ds/dt(C) = 0) com a parte dianteira (pd(C)) muito proxima de um muro (M tal que dist(pd(C),M) eps, para algum eps suficientemente pequeno) e voce precisa sair dali (em t = t_0 0, temos que ter s(C)(t)

[obm-l] Re:[obm-l] TEOREMA DA DECOMPOSIÇÃ O PRIMÁRIA

2007-03-21 Por tôpico claudio.buffara
-- Cabeçalho original --- De: [EMAIL PROTECTED] Para: obm-l@mat.puc-rio.br Cópia: Data: Tue, 20 Mar 2007 21:24:01 -0300 (ART) Assunto: [obm-l] TEOREMA DA DECOMPOSIÇÃO PRIMÁRIA Desculpem-me. É TEOREMA DA DECOMPOSIÇÃO PRIMÁRIA. Obrigado Voce acha isso em varios lugares:

Re:[obm-l] desigualdade da Eureka romena (2)

2007-03-21 Por tôpico claudio.buffara
De:[EMAIL PROTECTED] Para:obm-l@mat.puc-rio.br Cópia: Data:Wed, 21 Mar 2007 13:00:32 + Assunto:[obm-l] desigualdade da Eureka romena (2) Sauda,c~oes, === 2(1+x^{n+1})^n = (1+x^n)^{n+1} para x0 , n\in N. === Tentei por indução e não consegui. Seja f:[0,+inf) - R dada por: f(x) =

Re:[obm-l] desigualdade da Eureka romena (2)

2007-03-21 Por tôpico claudio.buffara
Aí vai: Seja S_n(p) = S = 1 + 2^p + 3^p + n^p com n,p\in N; p = n 0. Mostre que [n/(p+1)] + 1/2 = S/n^p 2 . Fonte: Gazeta Matematica V.97, p.228. Compare Integral(0...1) x^p*dx com as somas de Riemann inferior e superior, usando n sub-intervalos de comprimento 1/n,

Re:[obm-l] soma da Eureka romena

2007-03-20 Por tôpico claudio.buffara
tan(a-b) = (tan(a)-tan(b))/(1+tan(a)*tan(b)) == tan(a)*tan(b) = (tan(a)-tan(b))/tan(a-b) - 1 a = (k+1)x e b = kx == tan((k+1)x)*tan(kx) = (tan((k+1)x) - tan(kx))/tan(x) - 1 == Soma(1=k=n-1) tan((k+1)x)*tan(kx) = Soma(1=k=n-1) ( (tan((k+1)x) - tan(kx))/tan(x) - 1 ) = (tan(nx) - tan(x))/tan(x) -

Re: [obm-l] Primos

2007-03-20 Por tôpico claudio.buffara
:30AM -0300, claudio.buffara wrote: ... Enfim, eu entrei no Google e digitei: primes congruent to 1 Dirichlet A terceira referencia foi: http://planetmath.org/encyclopedia/SpecialCaseOfDirichletsTheoremOnPrimesInArithmeticProgressions.html ... Estou com o seguinte problema: Para

Re:[obm-l] Primos

2007-03-19 Por tôpico claudio.buffara
Me desculpem se esta resposta parecer condescendente, mas uma das grandes vantagens da internet (talvez a maior, depois de pornografia gratis...rs) eh a facilidade com que obtemos informacoes que, sem ela, seriam praticamente inacessiveis (no caso presente, teriamos que ir a alguma biblioteca

Re:[obm-l] Casais

2007-03-17 Por tôpico claudio.buffara
k casais podem ser formados da seguinte maneira: -Escolha dos k rapazes: Binom(6,k) -Escolha das k garotas: Binom(10,k) -A primeira garota pode escolher dentre k rapazes; -A segunda dentre k-1; ... -A k-esima soh pode ficar com o rapaz que sobrou (mal-vestido, cheio de espinhas e com desodorante

Re:[obm-l] Casais

2007-03-16 Por tôpico claudio.buffara
-- Cabeçalho original --- De: [EMAIL PROTECTED] Para: obm-l@mat.puc-rio.br Cópia: Data: Thu, 15 Mar 2007 15:18:35 -0300 Assunto: [obm-l] Casais Problema No início de uma festa há 6 rapazes desacompanhados e 10 garotas desacompanhadas. Quantos são os estados possíveis no

Re:[obm-l] Feras

2007-03-16 Por tôpico claudio.buffara
Os feras da lista devem estar ocupados, mas a resposta a sua pergunta eh sim. Basta estender a definicao de Binom(x,k) para x nao inteiro (entretanto, com k inteiro). Binom(x,0) = 1, Binom(x,1) = x e Binom(x,k) = x*(x-1)*...*(x-k+1)/k! para k 1. Logo: Binom(1/2,0) = 1; Binom(1/2,1) = 1/2;

Re:[obm-l] Problema de algebra vetorial.

2007-03-14 Por tôpico claudio.buffara
Oi, Ronaldo: Chame de u o vetor de coordenadas (x1,y1,z1) e de v o segundo vetor. Gire os seus eixos coordenados de modo que as coordenadas de u passem a ser (0,0,L) e as de v (L*sen(theta),0,L*cos(theta)). Ou seja, u gera o novo eixo z e {u,v} gera o novo plano xz. Ao girar v de um angulo phi

[obm-l] Integral de Henstock-Kurzweil

2007-03-09 Por tôpico claudio.buffara
Eu nunca tinha ouvido falar desse negocio, mas parece ser interessante: http://www.math.vanderbilt.edu/~schectex/ccc/gauge/ Alem disso, alguns matematicos acham ateh que o curriculo de Calculo I deve ser reformado: http://www.math.vanderbilt.edu/~schectex/ccc/gauge/letter/ []s, Claudio.

[obm-l] Re:[obm-l] Re: [obm-l] Alguém pode me ajudar?

2007-03-02 Por tôpico claudio.buffara
Você conhece o teorema das médias potenciais (power means em inglês)? Ele diz que se x1, x2, ..., xn são inteiros positivos e a b (reais quaisquer), então: ((x1^a + ... + xn^a)/n)^(1/a) = ((x1^b + ... + xn^b)/n)^(1/b). (se a = 0 ou b = 0, então a média correspondente é a média geométrica)

[obm-l] Re:[obm-l] POLINÔMIOS

2007-03-02 Por tôpico claudio.buffara
De:[EMAIL PROTECTED] Para:obm-l@mat.puc-rio.br Cópia: Data:Thu, 1 Mar 2007 12:01:37 -0800 (PST) Assunto:[obm-l] POLINÔMIOS Quantas raízes reais têm os polinômios: a) x^3+3x^2+9x+9 A derivada é 3x^2 + 6x + 9 = 3(x+1)^2 + 6 0, para todo x. Logo, a função x - x^3 + 3x^2 + 9x + 9 é

[obm-l] Re: [obm-l] Re: [obm-l] USAMO - Soma tr igonométrica.

2007-03-02 Por tôpico claudio.buffara
De:[EMAIL PROTECTED] Para:obm-l@mat.puc-rio.br Cópia: Data:Thu, 1 Mar 2007 10:56:56 -0300 Assunto:Re: [obm-l] Re: [obm-l] USAMO - Soma trigonométrica. Vou aproveitar a soma trigonométrica e pedir novamente uma ajuda com o produto trigonométrico sen(kPI/n), k indo de 1 até n-1. Sei que o

Re:[obm-l] Corpo de caracteristica zero

2007-02-23 Por tôpico claudio.buffara
Seja K um corpo de caracteristica zero (ou seja, para todo n em N, 1_k+1_k+...+1_k 0_k (n parcelas)). K contem 0_k e 1_k, por definicao de corpo. Agora, se definirmos n_k = 1_k + 1_k + ... + 1_k (n parcelas), veremos que K contem uma copia de N. Alem disso, n_k em K == -n_k em K == K contem uma

Re:RES: [obm-l] Valor intermeio

2005-07-12 Por tôpico claudio.buffara
De: [EMAIL PROTECTED] Para: obm-l@mat.puc-rio.br Cópia: Data: Mon, 11 Jul 2005 11:11:16 -0300 Assunto: RES: [obm-l] Valor intermeio Aqui vale alguns comentários: 1) o centro de uma circunferência que passa por A e B está na mediatriz de A e B. Além disso, o raio dela

Re:[obm-l] Valor intermeio

2005-07-10 Por tôpico claudio.buffara
De: [EMAIL PROTECTED] Para: obm-l@mat.puc-rio.br Cópia: Data: Sun, 10 Jul 2005 12:41:57 +0200 Assunto: [obm-l] Valor intermeio 3- Dado um conjunto de 2n+3 pontos no plano de modo que não existam 4 que pertençam a mesma circunferencia, demostrar que existe uma circunferencia

[obm-l] Série Divergente

2005-06-28 Por tôpico claudio.buffara
Oi, pessoal: Achei esse problema interessante: Seja (a_n) uma sequência de termos positivos tal que a série SOMA(n=1) a_n diverge. Seja s_n = a_1 + a_2 + ... + a_n. Prove que SOMA(n=1) (a_n/s_n) também diverge. Isso prova que, dada uma série SOMA a_ndivergente de termos positivos, sempre

Re:[obm-l] series - conv. uniforme

2005-06-26 Por tôpico claudio.buffara
De: [EMAIL PROTECTED] Para: "obm-l" obm-l@mat.puc-rio.br Cópia: Data: Sun, 26 Jun 2005 17:24:11 -0300 Assunto: [obm-l] series - conv. uniforme Olá pessoal, encalhei neste aqui: Seja V := {z pert C | Im(z) 0 } Mostre que a serie de funcoes Somatoria[m = 1]

RE: RES: [obm-l] Oswald de Souza (off)

2005-06-23 Por tôpico claudio.buffara
De: [EMAIL PROTECTED] Para: obm-l@mat.puc-rio.br Cópia: Data: Thu, 23 Jun 2005 13:12:55 -0400 Assunto: RE: RES: [obm-l] Oswald de Souza (off) From: Artur Costa Steiner <[EMAIL PROTECTED]> [snip] Nesta lista mesmo hah exemplos disto. O Claudio Buffara, que conhece

Re: [obm-l] divisor

2005-06-23 Por tôpico claudio.buffara
que o Qwert disse a meu respeito em sua última mensagem. []s, Claudio. De: [EMAIL PROTECTED] Para: obm-l@mat.puc-rio.br Cópia: Data: Thu, 23 Jun 2005 16:46:53 -0300 Assunto: Re: [obm-l] divisor On Tue, Jun 21, 2005 at 03:50:14PM -0300, claudio.buffara wrote: On Mon, Jun 20

Re: [obm-l] Oswald de Souza (off)

2005-06-22 Por tôpico claudio.buffara
Concordo que a mídia propaga idéias distorcidas sobre vários assuntos. Por exemplo, eu trabalhei muitos anos no mercado financeiro e sempre me impressionei com os absurdos publicados por jornalistas supostamente especializados em jornais e revistas supostamente especializados. Tenho até medo de

Re: [obm-l] divisor

2005-06-21 Por tôpico claudio.buffara
De: [EMAIL PROTECTED] Para: obm-l@mat.puc-rio.br Cópia: Data: Tue, 21 Jun 2005 11:54:13 -0300 Assunto: Re: [obm-l] divisor On Mon, Jun 20, 2005 at 10:55:04PM -0300, fgb1 wrote: Pessoal, preciso de ajuda nessa: Um fator de 2^33 - 2^19 - 2^17 -1, entre 1000 e 5000 é:

Re:[obm-l] soma binomial

2005-06-14 Por tôpico claudio.buffara
De: [EMAIL PROTECTED] Para: obm-l@mat.puc-rio.br Cópia: Data: Mon, 13 Jun 2005 20:46:59 + Assunto: [obm-l] soma binomial Sauda,c~oes, Alguém saberia provar que \sum_{k=0}^n (3k-2n) \binom{n}{k}^2 \binom{2k}{k} = 0 ? []'s Luís Oi, Luís: Aqui vai minha

Re:[obm-l] Aplicacao do Teorema Chines

2005-05-31 Por tôpico claudio.buffara
De: [EMAIL PROTECTED] Para: obm-l@mat.puc-rio.br Cópia: Data: Tue, 31 May 2005 11:07:40 + Assunto: [obm-l] Aplicacao do Teorema Chines Ola Pessoal, A questao abaixo ja foi comentada e resolvida pelo pessoal da Lista, mas eu me lembrei de uma aplicacao interessante

[obm-l] Re:[obm-l] Valor máximo

2005-05-24 Por tôpico claudio.buffara
raiz(3^2+2^2) = raiz(13). Seja a tal que cos(a) = 3/raiz(13) e sen(a) = 2/raiz(13). Então: 3*sen(x) + 2*cos(x) = raiz(13)*((3/raiz(13))*sen(x) + (2/raiz(13))*cos(x)) = raiz(13)*(cos(a)*sen(x) + sen(a)*cos(x)) = raiz(13)*sen(x+a) = raiz(13), com igualdade sss sen(x+a) = 1. Logo, o valor máximo é

Re: [obm-l] Probleminha dos trancedentes.

2005-05-17 Por tôpico claudio.buffara
De: [EMAIL PROTECTED] Para: obm-l@mat.puc-rio.br Cópia: Data: Mon, 16 May 2005 18:07:11 -0300 (ART) Assunto: Re: [obm-l] Probleminha dos trancedentes. Em geral essas provas de transcendencia sao dificeis e usam bastante analise, Ou entao vc usa um resultado forte

Re:[obm-l] Transcendentes - forma definitiva.

2005-05-17 Por tôpico claudio.buffara
De: [EMAIL PROTECTED] Para: obm-l@mat.puc-rio.br Cópia: Data: Tue, 17 May 2005 15:47:30 + Assunto: [obm-l] Trancendentes - forma definitiva. Ola Pessoal desta lista ... OBM-L, Esse problema e antigo, bonito e foi proposto aqui nesta lista - se nao me falha a

[obm-l] Desenho do problema (era: Geometria Plana)

2005-05-17 Por tôpico claudio.buffara
De: [EMAIL PROTECTED] Para: obm-l@mat.puc-rio.br Cópia: Data: Tue, 17 May 2005 10:30:36 -0300 Assunto: Re: [obm-l] Geometria Plana Primeiro nosso heroi faz um desenho bem caprichado do problema, Oi, Nicolau: É permitido o uso de esquadros, compasso e/ou transferidor na

Re:[obm-l] Transcendentes - forma definitiva.

2005-05-17 Por tôpico claudio.buffara
aulo Santa Rita 3,2154,170505 From: "claudio.buffara" <[EMAIL PROTECTED]> Reply-To: obm-l@mat.puc-rio.br To: "obm-l" <OBM-L@MAT.PUC-RIO.BR> Subject: Re:[obm-l] Transcendentes - forma definitiva. Date: Tue, 17 May 2005 13:47:18 -0300 Ola Pessoal desta

Re: [obm-l] Probleminha dos trancedentes.

2005-05-16 Por tôpico claudio.buffara
De: [EMAIL PROTECTED] Para: obm-l@mat.puc-rio.br Cópia: Data: Mon, 16 May 2005 06:54:03 -0300 Assunto: Re: [obm-l] Probleminha dos trancedentes. Na verdade o polinomio tem que ter coeficientes inteiros, senao dado qualquer numero complexo qualquer a, ele eh raiz de p(x) = x

[obm-l] Re:[obm-l] Progressões

2005-05-10 Por tôpico claudio.buffara
De: [EMAIL PROTECTED] Para: obm-l@mat.puc-rio.br Cópia: Data: Mon, 9 May 2005 18:31:55 EDT Assunto: [obm-l] Progressões Olá, pessoal !1) Considere as progressões seguintes de n termos e calcule as somas indicadasa) (1 + 2 + 3 + ...)b) (1^2 + 2^2 + 3^2 + ...)c) (1^3 + 2^3 +

[obm-l] Irredutíveis e Anéis

2005-05-10 Por tôpico claudio.buffara
Preciso de ajuda como exercício 3 da seção IV.4 do livro Elementos de Álgebra (Arnaldo Garcia e Yves Lequain - Projeto Euclides): a) Mostre que Z[raiz(3)] é isomorfo a Z[x]/(x^2-3). b) Seja p um primo de Z. Mostre que p é um elemento primo de Z[raiz(3)] se e somente se o polinômio x^2 - 3 é

[obm-l] Re: [obm-l] Somatória

2005-05-10 Por tôpico claudio.buffara
De: [EMAIL PROTECTED] Para: obm-l@mat.puc-rio.br Cópia: Data: Tue, 10 May 2005 13:45:36 -0300 Assunto: Re: [obm-l] Somatória Citando Bruno Bonagura <[EMAIL PROTECTED]>: Acho que faz um ano que vi essa questão e jamais consegui responder. Sempre que tenho alguma idéia

[obm-l] Re:[obm-l] Questão de cálculo.

2005-05-10 Por tôpico claudio.buffara
No intervalo (3,4), (x-3)^2 x-3, de modo que não existemg eh nesse intervalo (contínuas ou não) com a propriedade mencionada. Tem certeza de que o enunciado é esse? Se você restringir g e h ao intervalo (-inf,3], então g(3) = h(3) = 0, de modo que não existe o quociente. Por outro lado,

Re:[obm-l] Grupo e Subgrupo - Ajuda

2005-05-10 Por tôpico claudio.buffara
De: [EMAIL PROTECTED] Para: obm-l@mat.puc-rio.br Cópia: Data: Tue, 10 May 2005 13:29:20 + Assunto: [obm-l] Grupo e Subgrupo - Ajuda Amigos, por favor me ajudem nessas duas questões: 1) Seja G um grupo abeliano finito no qual o número de soluções em G da equação x^n = e

[obm-l] Re: [obm-l] Progressão Ge ométrica.

2005-05-09 Por tôpico claudio.buffara
Acho que faltou dizer que o triângulo é retângulo (por exemplo). Nesse caso, os lados sao b, bq e bq^2 (b = medida do menor cateto e q é a razão, que deve ser maior do que 1). Pitágoras implica que 1 + q^2 = q^4 == q = raiz((1 + raiz(5))/2) Mais geralmente, se os lados forem 1 = q = q^2 e

[obm-l] Re:[obm-l] questão do Munkres

2005-05-09 Por tôpico claudio.buffara
De: [EMAIL PROTECTED] Para: "Lista OBM" obm-l@mat.puc-rio.br Cópia: Data: Mon, 9 May 2005 16:34:31 -0300 (ART) Assunto: [obm-l] questão do Munkres Gostaria que vocês dessa uma olhada se o problema abaixo, tirado do livro do James R. Munkres (Analysis on Manifolds) estah

Re: [obm-l] Mais Isomorfismos

2005-05-09 Por tôpico claudio.buffara
De: [EMAIL PROTECTED] Para: obm-l@mat.puc-rio.br Cópia: Data: Mon, 9 May 2005 09:59:14 -0300 Assunto: Re: [obm-l] Mais Isomorfismos On Fri, May 06, 2005 at 10:31:32PM -0300, Claudio Buffara wrote: on 06.05.05 17:22, Nicolau C. Saldanha at [EMAIL PROTECTED] wrote: On

[obm-l] Grupos de Quadrados

2005-05-04 Por tôpico claudio.buffara
1) Seja G um grupo finito não trivial. Prove que |G| é ímpar se e somente se, para todo x em G, existe y em G tal que x = y^2. 2) Seja G um grupo (não necessariamente finito) e H um subgrupo de G. Prove que se, para cada x em G,x^2 pertence a H então H é um subgrupo normal de G e o quociente G/H

Re: [obm-l] i^2 = -1 ??

2005-05-03 Por tôpico claudio.buffara
A falácia deve ser esta: -1 = i*i = raiz(-1)*raiz(-1) = raiz((-1)*(-1)) = raiz(1) = 1. Onde está o erro? (dica: não tem nada a ver com o fato de denotarmos raiz(-1) por i, de modo que as duas primeiras igualdades são perfeitamente válidas). []s, Claudio. De: [EMAIL PROTECTED] Para:

[obm-l] Banach-Tarski júnior

2005-05-03 Por tôpico claudio.buffara
Achei esse exercício bem legal: Defina uma relação R no intervalo [0,1] da seguinte forma: xRy == x - y é racional. 1) Prove que R é uma relação de equivalência, a qual particiona [0,1] numa infinidade não-enumerável de subconjuntos (classes de equivalência) enumeráveis, um dos quais consiste

Re: [obm-l] autovalores , autovetores

2005-05-02 Por tôpico claudio.buffara
De: [EMAIL PROTECTED] Para: obm-l@mat.puc-rio.br Cópia: Data: Mon, 02 May 2005 13:44:25 -0300 Assunto: Re: [obm-l] autovalores , autovetores Vamos passar a limpo. Os resultados seguintes sao do livro de R.A Johnson e Dean W. Wichern (Applied Multivariate Statistical

Re:[obm-l] autovalores , autovetores

2005-04-29 Por tôpico claudio.buffara
Oi, Niski: Estou supondo que u é um vetor coluna do R^n. Nesse caso, a matriz u*u' tem o elemento (i,j) igual a u(i)*u(j) (produto da i-ésima e j-ésima componentes de u). Ou seja, a i-ésima linha de u*u' é igual a u(i)*u. Logo, u*u' temposto 1 e, portanto, n-1 autovalores são iguais a 0.

[obm-l] não-nilpotente de um anel

2005-04-26 Por tôpico claudio.buffara
Acho que agora foi... O teorema é o seguinte: SejamA um anel comutativo com 1 ex um elemento não-nilpotente de A. Então A possui um ideal I ao qualx não pertence. Se A for um corpo, então qualquer não-nilpotente é invertível e nenhum invertível pertence ao ideal (0). Assim, podemos tomar I =

[obm-l] Re:[obm-l] não-nilpotente de um anel

2005-04-26 Por tôpico claudio.buffara
Caramba! Eu devo estar com alguma maldição. Não consigo acertarnada de primeira... O enunciado correto do teorema deveria ser: SejamA um anel comutativo com 1 que não é corpo ex um elemento não-nilpotente de A. Então A possui um ideal I (0) ao qualx não pertence. []s, Claudio. De: [EMAIL

[obm-l] Re:[obm-l] Re: [obm-l] IME - Função

2005-04-20 Por tôpico claudio.buffara
Do jeito que está escrita não é injetora pois: h(x,0) = h(x,a) = (x^3,x - f(x)) para qualquer a real. Também não é sobrejetora. Basta tomar algum b tal que f(b) b. Quem é a pré-imagem de (b^3,0)? E se f(x) = x para todo x real, quem é a pré-imagem de (0,1)? []s, Claudio. De: [EMAIL

Re: [obm-l] trigonometria-ajuda

2005-04-20 Por tôpico claudio.buffara
Nossa! Vocês são muito sofisticados pra mim. Acho mais fácil ver que p(z) = z^3 - 3z - 1 tem 3 raízes reais pois: p(-2) = -3 0 p(-1) = 1 0 == logo, temos uma raiz entre -2 e -1. p(0) = -1 == mais uma raiz, desta vez entre -1 e 0 p(1) = -3 p(2) = 1 0 == a terceira raiz está entre 1 e 2. []s,

Re: [obm-l] Elemento Nilpotente

2005-04-18 Por tôpico claudio.buffara
contradição, pois P é um ideal primo ao qual x não pertence. Logo, x é nilpotente. []s Carol - Original Message - From: claudio.buffara To: obm-l Sent: Sunday, April 17, 2005 7:12 PM Subject: [obm-l] Elemento Nilpotente Como é que eu provo que se um elemento pertence a todos os ideais

[obm-l] Elemento Nilpotente

2005-04-17 Por tôpico claudio.buffara
Como é que eu provo que se um elemento pertence a todos os ideais primos de um anel comutativo, então este elemento é nilpotente? Obs: a recíproca também vale, mas esta eu consegui provar. []s, Claudio.

[obm-l] Somas e Diferencas de Cubos

2005-04-15 Por tôpico claudio.buffara
Aqui vai um do livro do Diofantos: Prove que todointeiro positivoque pode ser expresso comoa diferenca dos cubos de dois racionaispositivos pode tambem ser expresso como a soma dos cubos de dois racionais positivos. Assim, existem racionais positivos a, b tais que 7 = 2^3 - 1^3 = a^3 + b^3. Quem

[obm-l] Dois bonitinhos de Anéis

2005-04-14 Por tôpico claudio.buffara
Sejam A e B anéis comutativos com 1 e A* e B* os respectivos grupos de invertíveis. 1) É fácil provar que se f:A - B é um homomorfismo, então f(A*) está contido em B*. Dê um exemplo onde esta inclusão é estrita. 2) Prove que A* não pode ter exatamente 5 elementos. []s, Claudio.

[obm-l] Re:[obm-l] Dois bonitinhos de Anéis

2005-04-14 Por tôpico claudio.buffara
Correção: no primeiro problema, é preciso supor que f(1) = 1. Infelizmente, mesmo supondo que f é sobrejetor, existe um exemplo razoavelmente óbvio que eu demorei pra ver. Ou seja, não é um problema tão bonitinho assim. O segundo é. []s, Claudio. De: [EMAIL PROTECTED] Para: "obm-l"

Re:[obm-l] ex.

2005-04-14 Por tôpico claudio.buffara
O sinal depende de se ele está indo em direção ao leste ou ao oeste. De: [EMAIL PROTECTED] Para: obm-l@mat.puc-rio.br Cópia: Data: Wed, 13 Apr 2005 14:01:42 -0300 Assunto: [obm-l] ex. Um corpo está suspenso numa balança de mola num navio que viaja ao longo do equador com

[obm-l] Re: [obm-l] Soluções inteiras ..

2005-04-14 Por tôpico claudio.buffara
a e b sao primos entre si. Se mdc(a,a^3+b^3) não for 1, vai existir um primo p que divide este mdc. Logo, p divide a e p divide a^3 + b^3 == p divide b^3 == p divide b == p divide mdc(a,b) == contradição. []s, Claudio. De: [EMAIL PROTECTED] Para: obm-l@mat.puc-rio.br Cópia:

[obm-l] Probabilidade (Gnedenko)

2005-04-14 Por tôpico claudio.buffara
Um ponto está em AB, chamemos de P,outro em BC, chamemos de Q. As linhas de interesse são AP ,PQ e QC. Qual a probabilidade de podermos formar um triangulo com essas três linhas. Lembrando que o comprimento de AB é a e o comprimento de BC é b. Suponhamos que ABC = t (0 t Pi) |AP| = a-x, |QC|

Re: [obm-l] Retorno a origem

2005-04-13 Por tôpico claudio.buffara
Existem diversas variações, todas baseadas num triangulo isósceles ABC, com |AB=|AC| e BAC = 20 graus. Algumasvocê obtem resolvendoo problema da voltaà origem com n = 9. Outra (clássica) seria a seguinte: Sejam D em AB e E em AC tais que BCD = 50 graus e CBE = 60 graus. Determine BED. Sobre a

Re: [obm-l] incoveniente!

2005-04-13 Por tôpico claudio.buffara
Certamente você está cometendo uma agressão. A vítima é a língua portuguesa: a grafia correta é "incoNveniente" - note o N entre o "o" e o "v". Quanto ao problema, minha sugestão é usar o referencial do ponteiro das horas, em relação ao qual o ponteiro dos minutos se desloca a uma velocidade

Re: [obm-l] exercicio de probabilidade do gnedenko ( dificil)

2005-04-13 Por tôpico claudio.buffara
Será que não é pra escolher dois pontos ao acaso no segmento AB? Nesse caso, chamando o ponto escolhido mais próximo de A de P e o outro de Q, o problema é achar a probabilidade de segmentos de comprimentos |AP|, |PQ| e |QB| formarem um triângulo. Se for isso, sugiro que se faça |AB| = 1, |AP| =

Re: [obm-l] Primos de Dirichlet da forma an + b...

2005-04-13 Por tôpico claudio.buffara
Ou, se você não tem acesso a estes livros, dê uma olhada em: http://www.maths.ex.ac.uk/~rjc/rjc.html []s, Claudio. De: [EMAIL PROTECTED] Para: obm-l@mat.puc-rio.br Cópia: Data: Wed, 13 Apr 2005 22:57:05 + Assunto: Re: [obm-l] Primos de Dirichlet da forma an + b...

[obm-l] Re:[obm-l] Soluções inteiras ..

2005-04-13 Por tôpico claudio.buffara
De: [EMAIL PROTECTED] Para: obm-l@mat.puc-rio.br Cópia: Data: Wed, 13 Apr 2005 21:18:06 -0300 Assunto: [obm-l] Soluções inteiras .. Olá pessoal da lista ! Estou com problemas pra resolver essa questão : Quais sãos as soluções inteiras da seguinte equação : x^3 + y^3 =

[obm-l] Re:

2005-04-13 Por tôpico claudio.buffara
O ângulo ABC é dado? De: [EMAIL PROTECTED] Para: obm-l@mat.puc-rio.br Cópia: Data: Wed, 13 Apr 2005 22:09:37 -0300 (ART) Assunto: Olá Claúdio. Essa sua dica é ótima se fosse o caso de os dois pontos estarem sobre o mesmo segmento de reta ( esse exercício está no

Re:[obm-l] series , convergencia uniforme

2005-04-12 Por tôpico claudio.buffara
Oi, Niski: Pra facilitar um pouco as contas, vou mudar de variáveis para: SOMA(m=0...inf) ((z-1)/z)^m, de modo que a n-ésima soma parcial é: S_n(z) = SOMA(m=0...n-1) ((z-1)/z)^m = (1 - ((z-1)/z)^n)/(1 - (z-1)/z) = z*(1 - ((z-1)/z)^n) Nesse caso, V = {z em C | |(z-1)/z| 1} = {z em C | Re(z)

Re: [obm-l] Somatorio com Serie de Fourier

2005-04-11 Por tôpico claudio.buffara
Acho queé isso mesmo. Pra mim,o problemaé provar que: se n é inteiro positivo e w = exp(i*2*pi/n), então: 1 + w + w^4 + w^9 + ... + w^((n-1)^2) = K(n)*raiz(n) onde K(n) = 1+i, 1, 0, i se n == 0, 1, 2, 3 (mod 4), respectivamente. Não me parecemuito trivial... Aliás, alguém conhece alguma

Re: [obm-l] Somatorio com Serie de Fourier

2005-04-11 Por tôpico claudio.buffara
Eu consegui provar o caso N == 2 (mod4), o qual,obviamente, deve ser o mais fácil... Sejam N = 4m+2 e w = cis(2*pi/N) = cis(pi/(2m+1)) == w^(2m+1) = -1. Olhando mod 4m+2: (2m+1)^2 = 4m^2+4m+1 == 2m+1 Logo, para 0 = k = 2m: (2m+1+k)^2 - k^2 = (2m+1)^2 + 2(2m+1)k == (2m+1)^2 == 2m+1 ==

Re: [obm-l] soma de termos

2005-04-06 Por tôpico claudio.buffara
Oi, Bernardo: Eu falei mal da indução porque acho queela produz demonstrações feias e sem-graça, apesar de em muitos casos, ser a única forma (conhecida) de se demonstrar algum resultado. Mas não é o caso da lei das colunas: C(k,k) + C(k+1,k) + C(k+2,k) + ... + C(n,k) = C(n+1,k+1). Imagine que

[obm-l] [x^n] == n (mod 2)

2005-04-06 Por tôpico claudio.buffara
Aqui vai um bonitinho: Ache um número real x tal que, para todo n inteiro e positivo, [x^n] tem a mesma paridade que n. [a] = maior inteiro que é menor ou igual a a. Se não me engano, há algum tempo, o Shine exibiu um y tal que [y^n] é sempre ímpar. []s, Claudio.

Re: [obm-l] soma de termos

2005-04-06 Por tôpico claudio.buffara
)] Binom(N-n,n) 2^(N-2n-1). []'s Luís From: "claudio.buffara" <[EMAIL PROTECTED]> Reply-To: obm-l@mat.puc-rio.br To: "obm-l" <OBM-L@MAT.PUC-RIO.BR> Subject: Re: [obm-l] soma de termos Date: Wed, 6 Apr 2005 09:01:29 -0300 Oi, Bernardo: Eu falei

[obm-l] Re:[obm-l] Quadrados mágicos

2005-04-06 Por tôpico claudio.buffara
Eu tenho a impressão de que a dimensão do espaço dos quadrados mágicos é n^2 - 2n + 1, pois, segundo o exercício 3.33 do Elon, os funcionais lineares de F^(2n) - F: L_1, L_2, ..., L_n, C_1, C_2, ..., C_(n-1), T eS onde: L_i = soma dos elementos da i-ésima linha; C_j = soma dos elementos da

[obm-l] Quadrado Mágico

2005-04-06 Por tôpico claudio.buffara
Acho que sei como demonstrar que L_i (1=i=n), C_j (1=j=n-1), T e S são funcionais lineares L.I. Suponhamos que existam escalres a_i (1=i=n), b_j (1=j=n), c e d tais que o funcional linear: F = SOMA(1...n) a_i*L_i + SOMA(1...n-1) b_j*C_j + c*T + d*S seja identicamente nulo. Seja A(i,j) a matriz

[obm-l] 1^2 + 2^2 + ... + n^2

2005-04-05 Por tôpico claudio.buffara
Ontem alguém perguntou aqui na lista como se demonstrava a fórmula da soma dos quadrados dos primeiros n inteiros positivos. Eu diria que 99% das pessoas usaria indução, o que além de ser mecânico e sacal, não ilustra o que realmente ocorre no problema e, o que é pior, se a fórmula não for

Re:[obm-l] dois dificeis de probabilidade

2005-04-05 Por tôpico claudio.buffara
De: [EMAIL PROTECTED] Para: obm-l@mat.puc-rio.br Cópia: Data: Tue, 5 Apr 2005 21:07:02 -0300 (ART) Assunto: [obm-l] dois dificeis de probabilidade Não estou conseguindo fazer estes dois. Qualquer ajuda será bem vinda: 1) Há um fila de cinemacom 2n pessoas. O cinema custa

Re:[obm-l] Geometria

2005-04-04 Por tôpico claudio.buffara
É fácil ver que, para qualquer P no interior do triângulo, vale: ax + by + cz = 2A, onde A = área do triangulo. Você quer o valor mínimo de F(x,y,z) = a/x + b/y+ c/z. 2A*F(x,y,z) = (ax + by + cz)*(a/x + b/y + c/z) = a^2 + b^2 + c^2 + ab(x/y + y/x) + ac(x/z + z/x) + bc(y/z + z/y) = a^2 + b^2 +

Re:[obm-l] Problema do Kuratowski

2005-04-03 Por tôpico claudio.buffara
Oi, Paulo (e quem mais estiver interessado): Achei uma solução pra esse problema aqui: http://www.mctague.org/carl/fun/kuratowski/kuratowski.pdf Umconjunto que gera 14 conjuntos distintos é: (0,1) união (1,2) união [Q inter (3,4)] união {5}. E uma generalização aqui:

[obm-l] Re:[obm-l] RE: [obm-l] Re:[obm-l] DEmonstração Mais elementar.

2005-04-03 Por tôpico claudio.buffara
: Sun, 03 Apr 2005 11:56:49 -0300 Assunto: [obm-l] RE: [obm-l] Re:[obm-l] DEmonstração Mais elementar. Olá Cláudio. está aí o nó da questão. Não conheço demonstração de que 1/p seja dízima periódica simples que não use o Peq. teorema... Um abraço, Frederico. From: "claudio.bu

[obm-l] Re: [obm-l] Matrizes invertíveis....

2005-04-03 Por tôpico claudio.buffara
Pra mostrar que o conjunto das matrizes ortogonais é fechado, você poderia também mostrar que o seu complementar M é aberto. A pertence a M == A'A I. A função F: R^(n^2) x R^(n^2) - R^(n^2) dada por F(X) = X'X é contínua e M é a imagem inversa por F do aberto R^(n^2) - {I}. []s, Claudio.

[obm-l] Re:[obm-l] DEmonstração Mais elementar.

2005-04-02 Por tôpico claudio.buffara
Se p = 3, então p divide 111, 11, 1, e qualquer número formado por 3k algarismos 1 (k inteiro positivo). Suponhamos, portanto, que p 2, 3 e5. Nesse caso, 1/p é uma dízima periódica simples (não sei se isso é mais fácil de demonstrar do que o pequeno teorema de Fermat ou o teorema de

Re:[obm-l] Problema do Kuratowski

2005-04-02 Por tôpico claudio.buffara
Oi, Paulo: Imagino que o que você queira é gerar, a partir de A_1 = A, por sucessivas aplicações de F ou C, uma sequência de conjuntos A_1, A_2, ..., tal que: i)A_(n+1) = F(A_n) ou A_(n+1) = C(A_n) e ii)a família {A_1, A_2, ...} tenha a maior cardinalidade possível. Eu acho que a maior

  1   2   3   >